Search found 2095 matches


When we're asked what assertion is supported, we're really asked what MUST be true, i.e. what's inferable. If the 2 lemonade stands reported identical revenues and identical profits, we can infer 2 things: 1. the costs must have been identical , since Profit = Revenue - Costs . 2. Since revenue (sim...


First, infer everything we can from the given information: \(w, x\) and \(y\) are positive integers such that \(w \leq x \leq y\) the average (arithmetic mean) of \(w, x\) and \(y\) is \(20\) Infer --> the sum of w, x, and y is 60. Question: is \(w > 15\)? Since we know that the sum is 60 and w is t...

by ceilidh.erickson

Fri Apr 26, 2019 11:18 am
Forum: Data Sufficiency
Topic: \(w, x\) and \(y\) are positive integers such that
Replies: 1
Views: 414

This question - like many others from Veritas Prep - directly plagiarizes an Official Guide question: CR # 87, OG 13/2015: In the United States, of the people who moved from one state to another when they retired, the percentage who retired to Florida has decreased by three percentage points over th...


If 10 issues cost $24, that represents an individual cost of $2.40 each.

If $2.40 is 20% less than the cover price x, then:
$$2.40=\left(0.8\right)x$$
$$2.40=\left(\frac{4}{5}\right)x$$
$$2.40\left(\frac{5}{4}\right)=x$$
$$0.60\left(5\right)=x$$
$$3.00=x$$

The answer is E.


To count the number of possible crews, we must calculate 2 things: 1. the number of ways to select 1 person with experience out of 12 2. the number of ways to select the remaining 2 people out of the remaining 9 Remember that when counting combinations in a diminishing pool in which order doesn't ma...


For more on how the GMAT subverts our expectations of the 2 equations / 2 variables rule, see:
https://www.manhattanprep.com/gmat/blog ... ons-rules/

by ceilidh.erickson

Sat Apr 13, 2019 4:34 pm
Forum: Data Sufficiency
Topic: What is the value of xy?
Replies: 2
Views: 578

Question: What is the value of xy? To answer this question, we need either values for x and y individually, or simply a value for the product xy. (1) y = x + 1 This gives us y in terms of x, but cannot give us the value of xy. Insufficient. (2) y = x^2 + 1 This gives us y in terms of x^2, but canno...

by ceilidh.erickson

Sat Apr 13, 2019 4:22 pm
Forum: Data Sufficiency
Topic: What is the value of xy?
Replies: 2
Views: 578

Remainder problems in DS often require TESTING CASES: If the positive integer n is greater than 6, what is the remainder when n is divided by 6? We don't have much given information to unpack here, so let's move on to the statements: 1) When n is divided by 9, the remainder is 2. Translate: n = (som...


edit

Probability is always defined this way: probability = (# of desired outcomes)/(total possible # of outcomes) You correctly identified that there was only one pairing of terms that would give you a difference of squares: (x + y) and (x - y). However, you miscounted your total # of possible outcomes. ...

by ceilidh.erickson

Thu Apr 04, 2019 11:25 am
Forum: GMAT Math
Topic: GMAT Prep Practice Test Quant Questions #3
Replies: 5
Views: 10297

In DS, the GMAT often likes to mix up PROPORTIONS v. REAL NUMBERS. Generally speaking, a real number won't help to answer a proportion question (unless you're give more info). Here are more in a similar vein to try: https://www.beatthegmat.com/by-what-percent-was-the-price-of-a-certain-television-se...

by ceilidh.erickson

Thu Apr 04, 2019 10:42 am
Forum: GMAT Math
Topic: GMAT Prep Practice Test Quant Questions #2
Replies: 2
Views: 3098

We can TEST CASES to prove insufficiency in this problem. Warehouse W’s revenue from the sale of sofas was what percent greater this year than it was last year? Revenue from sales will equal (price of sofas)(quantity sold) --> R = (p)(q) To answer the question of the proportional difference in rev...

by ceilidh.erickson

Thu Apr 04, 2019 10:23 am
Forum: GMAT Math
Topic: GMAT Prep Practice Test Quant Questions #2
Replies: 2
Views: 3098

Since negative exponents indicate the reciprocal of base to the positive exponent, we can rewrite our exponents and bases: $$\left(\frac{1}{5}\right)^M=5^{-M}$$ $$\left(\frac{1}{4}\right)^{18}=4^{-18}=2^{-36}$$ $$\frac{1}{\left(2\right)\left(10\right)^{35}}=(2^{-1})(10^{-35})=(2^{-1})((5\cdot2)^{-35...

by ceilidh.erickson

Thu Apr 04, 2019 10:11 am
Forum: GMAT Math
Topic: GMAT Prep Practice Test Quant Questions
Replies: 1
Views: 2824

If the train traveled for 25 hrs at 50 mph, it traveled a total of 1250 miles: (Rate)(Time) = Distance. Let's calculate the distance of each of the answer choices to town W. A. T --> V --> W = 600 + 750 = 1350 B. U --> V --> W = 500 + 750 = 1250 C. V --> W = 750 D. Y --> X --> W = 675 + 125 = 800 E....


Overlapping Sets are a common question type on the GMAT, and the double-set matrix is often the easiest way to solve. For more on how to use the matrix, see: https://www.beatthegmat.com/of-the-800-sweaters-in-a-certain-store-150-are-red-how-t306943.html#828510 https://www.beatthegmat.com/at-a-charit...


This is an OVERLAPPING SETS question. The easiest way to solve is by using a DOUBLE SET MATRIX: https://i.postimg.cc/1gW91b8Y/Screenshot-2019-04-04-13-42-19.png We can fill in what we know from the question stem: If 20 percent of all employees have one year or more of experience and no advanced degr...